Sequence of integrable functions (f_n) conv. to f

In summary, the sequence of integrable functions (f_n) converges to f if the limit of the integrals of f_n exists and is equal to the integral of f. This means that as n approaches infinity, the values of f_n become closer and closer to the values of f, resulting in a continuous and well-behaved function f. This convergence has important implications in the study of analysis and calculus, allowing for the evaluation of complex integrals and providing a useful tool for solving problems in mathematics and engineering.
  • #1
fishturtle1
394
82
Homework Statement
Let ##(f_n)## be a sequence of integrable functions on ##[a,b]##, and suppose ##f_n \rightarrow f## uniformly on ##[a,b]##. Prove that ##f## is integrable and
$$\int_a^b f = \lim_{n\rightarrow\infty} \int_a^b f_n$$
Relevant Equations
.
##\textbf{Attempt at solution}##: If I can show that ##f## is integrable on ##[a,b]##, then for the second part I get :

Let ##\frac{\varepsilon}{b-a} > 0##. By definition of uniform convergence, there exists ##N = N(\varepsilon) > 0## such that for all ##x \in [a,b]## we have ##\vert f(x) - f_n(x) \vert < \frac{\varepsilon}{b-a}##. This gives us,

$$\vert \int_a^b f(x)dx - \int_a^b f_n(x)dx \vert = \vert \int_a^b f(x) - f_n(x) dx \vert \le \int_a^b \vert f(x) - f_n(x) \vert dx < \int_a^b \frac{\varepsilon}{b-a}dx = \varepsilon$$ when ##n > N##.

It follows ##\lim_{n\rightarrow\infty} \int_a^b f_n(x) dx = \int_a^b f(x) dx## for all ##x \in [a,b]##. []

For the first part I am stuck. Let ##\varepsilon > 0##. Then I need a partition ##P## of ##[a,b]## such that ##U(f, P) - L(f, P) < \varepsilon##. We're given that for any ##n##, there is a partition ##P## such that ##U(f_n, P) - L(f_n, P) < \varepsilon##. How can I proceed?
 
  • Like
Likes Delta2
Physics news on Phys.org
  • #2
fishturtle1 said:
For the first part I am stuck. Let ##\varepsilon > 0##. Then I need a partition ##P## of ##[a,b]## such that ##U(f, P) - L(f, P) < \varepsilon##. We're given that for any ##n##, there is a partition ##P## such that ##U(f_n, P) - L(f_n, P) < \varepsilon##. How can I proceed?

The upper and lower sums work like integrals in a way, so you can use a similar technique for these as you used to show part b).
 
  • Informative
Likes fishturtle1
  • #3
Here is a measure theoretic solution which you may appreciate later (maybe you treat this theorem in a first course, probably not).

We use the following result (see any text on measure theory or more advanced texts on Riemann-integration).
A function is Riemann-integrable if and only if it is bounded and its set of discontinuities has (Lebesgue) measure ##0##.

The points where atleast one ##f_n## is discontinuous has measure zero so the limit of the ##f_n's## is almost surely continuous as uniform convergence preserves continuity. Hence the limit is integrable.
 
Last edited by a moderator:
  • Like
Likes fishturtle1
  • #4
PeroK said:
The upper and lower sums work like integrals in a way, so you can use a similar technique for these as you used to show part b).
We'll show ##f## is integrable on ##[a,b]##.

Proof: Let ##\frac{\varepsilon}{4(b-a)} > 0##. Then there is ##N = N(\frac{\varepsilon}{4(b-a)}) > 0## such that for ##x \in [a,b]## and ##n > N## implies $$\vert f_n(x) - f(x) \vert < \frac{\varepsilon}{4(b-a)}$$ i.e
$$-\frac{\varepsilon}{4(b-a)} < f(x) - f_n(x) < \frac{\varepsilon}{4(b-a)} $$
Fix ##n > N##. For ##\frac{\varepsilon}{2} > 0##, there exists partition ##P = \lbrace a = t_0 < t_1 < \dots < t_k = b \rbrace## such that ##U(f_n, P) - L(f_n, P) < \frac{\varepsilon}{2}##.

Now, $$U(f - f_n, P) = \sum_{i=1}^{k} M(f - f_n, [t_{i-1}, t_i]) \cdot (t_i - t_{i-1}) < \sum_{i=1}^{k} \frac{\varepsilon}{4(b-a)} \cdot (t_i - t_{i-1}) = \frac{\varepsilon}{4(b-a)} \sum_{i=1}^{k} (t_i - t_{i-1}) = \frac{\varepsilon}{4}$$

Also, $$L(f - f_n, P) = \sum_{i=1}^{k}m(f - f_n, [t_{i-1}, t_i])\cdot (t_i - t_{-1}) > \sum_{i-1}^{k} -\frac{\varepsilon}{4(b-a)}\cdot(t_i - t_{i-1}) = - \frac{\varepsilon}{4}$$

Thus, $$U(f, P) - L(f, P) \le U(f - f_n, P) + U(f_n, P) - (L(f - f_n, P) + L(f_n, P)) < \varepsilon$$
It follows that ##f## is integrable on ##[a,b]##. []
 
Last edited:
  • Like
Likes Delta2
  • #5
Math_QED said:
Here is a measure theoretic solution which you may appreciate later (maybe you treat this theorem in a first course, probably not).

We use the following result (see any text on measure theory or more advanced texts on Riemann-integration.The points where atleast one ##f_n## is discontinuous has measure zero so the limit of the ##f_n's## is almost surely continuous as uniform convergence preserves continuity. Hence the limit is integrable.
Thanks for posting this. I think I am learning about this, this upcoming semester.
 
  • Like
Likes member 587159
  • #6
fishturtle1 said:
Thanks for posting this. I think I am learning about this, this upcoming semester.

It's a very powerful tool and actually weird it doesn't occur in much places. For example, it immediately implies that continuous functions, monotonic functions, step-functions, functions with countably many discontinuities (e.g. https://en.wikipedia.org/wiki/Thomae's_function) are all Riemann-integrable. It can also be used to quickly show that something is not Riemann-integrable. For example, an indicator function on the rationals or the Cantor set can not be Riemann-integrable (note that these become integrable if one considers the Lebesgue integral instead of the Riemann-integral).
 
  • Informative
Likes fishturtle1

1. What is the definition of the sequence of integrable functions (f_n) conv. to f?

The sequence of integrable functions (f_n) conv. to f means that as n approaches infinity, the integral of each function in the sequence converges to the integral of the final function f. In other words, the sequence of functions becomes closer and closer to the final function as n increases.

2. How is the convergence of the sequence of integrable functions (f_n) conv. to f determined?

The convergence of the sequence of integrable functions (f_n) conv. to f is determined by the limit of the integral of each function in the sequence as n approaches infinity. If this limit exists and is equal to the integral of the final function f, then the sequence is said to converge.

3. What is the importance of the sequence of integrable functions (f_n) conv. to f in mathematics?

The sequence of integrable functions (f_n) conv. to f is important in mathematics because it allows for the study of convergence of functions. This is particularly useful in the field of analysis, where the convergence of sequences of functions is a fundamental concept.

4. Can the sequence of integrable functions (f_n) conv. to f converge to a function that is not integrable?

No, the sequence of integrable functions (f_n) conv. to f can only converge to a function that is also integrable. This is because the integral of a function is a measure of its size, and if the integral of the final function f is finite, then the integrals of the functions in the sequence must also be finite.

5. Are there any conditions that must be satisfied for the sequence of integrable functions (f_n) conv. to f to converge?

Yes, there are certain conditions that must be satisfied for the sequence of integrable functions (f_n) conv. to f to converge. These include the functions in the sequence being bounded and the sequence being uniformly convergent. These conditions ensure that the limit of the integral of each function in the sequence exists and is equal to the integral of the final function f.

Similar threads

  • Calculus and Beyond Homework Help
Replies
6
Views
2K
  • Calculus and Beyond Homework Help
Replies
8
Views
669
  • Calculus and Beyond Homework Help
Replies
4
Views
319
  • Calculus and Beyond Homework Help
Replies
7
Views
1K
  • Calculus and Beyond Homework Help
Replies
12
Views
1K
  • Calculus and Beyond Homework Help
Replies
15
Views
1K
  • Calculus and Beyond Homework Help
Replies
7
Views
2K
  • Calculus and Beyond Homework Help
Replies
13
Views
967
  • Calculus and Beyond Homework Help
Replies
9
Views
1K
  • Calculus and Beyond Homework Help
Replies
9
Views
1K
Back
Top